Difference between revisions of "1985 AJHSME Problems/Problem 1"

(Solution 1)
m (Solution 2 (Brute force))
Line 9: Line 9:
  
 
==Solution 2 (Brute force)==
 
==Solution 2 (Brute force)==
 
+
Multlipication gives:<cmath>\frac{15}{99\cdot\frac{693}{105}=\frac{10395}{10395}=\boxed{\text{(A)}\ 1}.</cmath>
If you want to multiply it out, then it would be <cmath>\frac{15}{99} \cdot \frac{693}{105}= \frac{10395}{10395}= \boxed{\text{(A)}\ 1}.</cmath>
 
  
 
==See Also==
 
==See Also==

Revision as of 23:44, 12 March 2023

Problem

$\frac{3\times 5}{9\times 11}\times \frac{7\times 9\times 11}{3\times 5\times 7}=$

$\text{(A)}\ 1 \qquad \text{(B)}\ 0 \qquad \text{(C)}\ 49 \qquad \text{(D)}\ \frac{1}{49} \qquad \text{(E)}\ 50$

Solution 1

By the associative property, we can rearrange the numbers in the numerator and the denominator. \[\frac{3}{3}\cdot \frac{5}{5}\cdot\frac{7}{7}\cdot\frac{9}{9}\cdot\frac{11}{11}=1\cdot1\cdot1\cdot1\cdot1=\boxed{\text{(A)} 1}\]

Solution 2 (Brute force)

Multlipication gives:

\[\frac{15}{99\cdot\frac{693}{105}=\frac{10395}{10395}=\boxed{\text{(A)}\ 1}.\] (Error compiling LaTeX. Unknown error_msg)

See Also

1985 AJHSME (ProblemsAnswer KeyResources)
Preceded by
First
Question
Followed by
Problem 2
1 2 3 4 5 6 7 8 9 10 11 12 13 14 15 16 17 18 19 20 21 22 23 24 25
All AJHSME/AMC 8 Problems and Solutions


The problems on this page are copyrighted by the Mathematical Association of America's American Mathematics Competitions. AMC logo.png